A significant amount of the acquisition budget of a typical university library is spent on subscriptions to scholarly...

shafieiava on March 19, 2020

Approach to question and A-E

Can someone explain how they arrived at the correct answer? An explain of answer choices A-E would be particularly helpful here. I had a hard time narrowing down the answer choices.

Reply
Create a free account to read and take part in forum discussions.

Already have an account? log in

Annie on March 19, 2020

Hi @shafieiava,

This question is asking you to find the answer choice which most weakens the argument. The best way to start with a question like this is to break down the argument so you understand it:

Premise: A large part of library budgets goes to buying journals.
Premise: The average price of journals has gone up, even though the cost to make them hasn't changed.
Conclusion: Therefore, most journals must be making more money now.

You're looking for an answer choice that undermines the conclusion or the connection between the conclusion and the premises.

Answer Choices:
(A) is incorrect because it is irrelevant. The argument is about the profit being made by journals but this answer choice is about how libraries are paying for journals.

(B) is incorrect because it is irrelevant. Again, the argument is about the money journals are making, not about university budgets.

(C) is incorrect but tricky. It is relevant because it is talking about the cost of publishing a journal, which directly relates to how much money a journal is making. But, the argument is about how journal prices have risen over the last several years and how this means there has been a change in their profit. This answer choice is not about a change in profit.

(D) is correct because it directly counters the conclusion. The argument says that because the prices journals charge libraries have gone up, the journals must be making a lot more money. But, this answer choice tells us that libraries aren't actually a bit customer for journals. Therefore, the idea that the journals are making a lot more money is called into question.

(E) is incorrect because it is irrelevant. The number of times a journal is published per year, if consistent (and we have no reason to believe it is not consistent) doesn't affect its profit.